yama_sekander
Thanks Received: 4
Vinny Gambini
Vinny Gambini
 
Posts: 24
Joined: January 16th, 2011
 
 
 

Q14 - Yolanda: Gaining access to computers

by yama_sekander Tue Jul 19, 2011 11:38 pm

i narrowed it down between A and C and i am wondering why A is wrong and C is right.

in my mind it seemed like the flaw had to do with an improper use of analogy. hence, i thought that Arjun did not maintain that specific distinction in his argument.


also, how is presenting a possible scenario a flaw?
User avatar
 
ManhattanPrepLSAT1
Thanks Received: 1909
Atticus Finch
Atticus Finch
 
Posts: 2851
Joined: October 07th, 2009
 
This post thanked 2 times.
 
 

Re: Yolanda: Gaining access to computers

by ManhattanPrepLSAT1 Tue Jul 26, 2011 11:42 pm

The flaw in Arjun's reasoning is that he has mistaken something that could be true for something that is true. Notice the "could" in the evidence. This is exactly the same as PT45, S4, Q14

As for answer choice (A), it is true that Arjun does not maintain the distinction made by Yolanda, but that's okay. Just because the answer choice describes something occurring in the argument, doesn't mean that it describes a flaw committed in the argument.

Yolanda's conclusion is that joyriding is the more dangerous crime, since it presents physical danger to people whereas computer crimes do not. Why would it be a flaw for Arjun to point out that Yolanda's argument was based on evidence that may not be true?

So let's just do a recap...

Arjun says that because unauthorized use of medical records systems in hospitals could present a danger to the lives of humans, that it does in fact present a danger to the lives of humans.

This argument mistakes something that is possible for something that is inevitable - best expressed in answer choice (C).

(A) is true, but does not represent an error in reasoning on the part of Arjun.
(B) is not true. Arjun does provide evidence.
(D) is not true. There is no confusing sufficiency with necessity. There are no language cues implying that we should focus on conditionality.
(E) is not true. Arjun's evidence is consistent with his conclusion, but is insufficient to ensure it.

Hope that helps!
 
auctiontheory
Thanks Received: 0
Forum Guests
 
Posts: 2
Joined: May 22nd, 2013
 
 
 

Re: Q14 - Yolanda: Gaining access to computers

by auctiontheory Wed May 22, 2013 8:32 am

mattsherman Wrote:The flaw in Arjun's reasoning is that he has mistaken something that could be true for something that is true. Notice the "could" in the evidence. This is exactly the same as PT45, S4, Q14

As for answer choice (A), it is true that Arjun does not maintain the distinction made by Yolanda, but that's okay. Just because the answer choice describes something occurring in the argument, doesn't mean that it describes a flaw committed in the argument.


I have a question about this. First of all, can you tell me what the "distinction" is? I did go for A, but only because I didn't think any other answer seemed right. I guess the distinction isn't made, but I'm not sure what it is.

Second, IMO, Arjun's conclusion is not crystal-clear. He simply says that he disagrees, but it's not certain what part of Yolanda's argument he's disagreeing with.

If he's disagreeing with Y's conclusion in that joyriding is NOT the more dangerous crime compared to unauthorized use of computers, then C would be the answer. But I think it can also be true that he might not be disagreeing with Y's conclusion but rather questioning her premise. i.e. I disagee (with your premise)! Y says that since joyriding endangers people, and computer crimes don't, joyriding is more dangerous. Notice how she says that in the case of computer crimes, ONLY intellecutal property is harmed(presumably, nothing else is harmed). This is a pretty extreme premise, IMO, and gives a reason for Arjun to disagree.

If this is the case, I don't see how C could be the answer. If Arjun is disagreeing with Y's premise, he need not prove that something actually occured. He can simply reason that there are ways in which people can be harmed in addition to intellectual property. Can anyone tell me why this line of reasoning would be invalid? (Which I assume so because C is correct) Thanks!!
 
sabrina_ashlie
Thanks Received: 1
Forum Guests
 
Posts: 1
Joined: June 04th, 2013
 
This post thanked 1 time.
 
 

Re: Q14 - Yolanda: Gaining access to computers

by sabrina_ashlie Sun Sep 01, 2013 12:09 am

I'm not getting why the distinction doesn't matter. As far as I can tell, Yolanda concludes that joyriding is MORE dangerous than computer crimes. Arjun's response would have us believing that computer hacking is also dangerous, but it doesn't address Yolanda's contention that joyriding is MORE dangerous, it just proves that, in some scenarios, hacking is dangerous too. To me, he is not addressing the root of the issue here.

I may be alone on this, but I am not seeing that C is a better answer than A.
User avatar
 
WaltGrace1983
Thanks Received: 207
Atticus Finch
Atticus Finch
 
Posts: 837
Joined: March 30th, 2013
 
This post thanked 2 times.
 
trophy
Most Thanked
trophy
Most Thankful
trophy
First Responder
 

Re: Q14 - Yolanda: Gaining access to computers

by WaltGrace1983 Mon Jan 27, 2014 5:07 pm

I think I see what is going on here. I picked (A) too and eliminated (C).

I think the "I disagree" is actually not important in terms of what it is referring to: premise or conclusion, either way I think the following analysis works.

Arjun says that the unauthorized use of medical records systems could damage data systems on which lives depend and therefore they also cause physical harm to people.

So look at what is happening. Arjun is saying that because this COULD happen then it DOES cause harm. If we put this into a simple argument then we may be able to see it clearer:

That Ferrari could kill me! → That Ferrari causes me harm.

It is taking something that is a possibility and making into something that is definitely true.

That Teddy Bear has the possibility of harming me (if I choke on it) → That Teddy Bear is harmful.

Is that correct?
User avatar
 
Mab6q
Thanks Received: 31
Atticus Finch
Atticus Finch
 
Posts: 290
Joined: June 30th, 2013
 
This post thanked 1 time.
 
 

Re: Q14 - Yolanda: Gaining access to computers

by Mab6q Fri Sep 26, 2014 8:36 pm

sabrina_ashlie Wrote:I'm not getting why the distinction doesn't matter. As far as I can tell, Yolanda concludes that joyriding is MORE dangerous than computer crimes. Arjun's response would have us believing that computer hacking is also dangerous, but it doesn't address Yolanda's contention that joyriding is MORE dangerous, it just proves that, in some scenarios, hacking is dangerous too. To me, he is not addressing the root of the issue here.

I may be alone on this, but I am not seeing that C is a better answer than A.


Arjun is not only disagreeing with the conclusion, but the argument that Yolanda makes: because joyriding physically endangers people --> it is more dangerous. He attacks Yolanda's argument by attempting to show that what is true of joyriding is also true of unauthorized use of medical records, and as such, that conclusion cannot be drawn for one and not the other. So, the issue is not the distinction.
"Just keep swimming"
 
keonheecho
Thanks Received: 0
Elle Woods
Elle Woods
 
Posts: 54
Joined: August 20th, 2014
 
 
 

Re: Q14 - Yolanda: Gaining access to computers

by keonheecho Sun Jun 26, 2016 6:17 pm

Based on question #13, can't the 'I disagree!' in Arjun's argument be inferred to refer to Yolanda's claim that joyriding is more dangerous than recklessly using private property?

If so, Arjun is refuting Yolanda's conclusion that joyriding is MORE dangerous. But Arjun only provides evidence that reckless use of private property is also dangerous. This doesn't provide evidence that one is not more dangerous than the other. So why can't (B) be correct?

Is it because even if irrelevant evidence is provided to deny an argument, it is still considered evidence against an argument?
User avatar
 
LolaC289
Thanks Received: 21
Elle Woods
Elle Woods
 
Posts: 92
Joined: January 03rd, 2018
 
This post thanked 1 time.
 
 

Re: Q14 - Yolanda: Gaining access to computers

by LolaC289 Fri Jul 20, 2018 4:58 am

The distinction Yolanda made is that computer crime is not as dangerous as joyride because computer crime does not cause physical harm while joyride does.

Arjun refutes Yolanda exactly by saying computer crime could also cause physical harm. He maintained the distinction Yolanda made, it's just that he didn't mention the word joyride in his argument.

Thus the flaw is simply with his own support given for his conclusion. He used something imaginary to try to establish a factual causal relationship.
 
MeenaV936
Thanks Received: 1
Jackie Chiles
Jackie Chiles
 
Posts: 33
Joined: February 16th, 2019
 
 
 

Re: Q14 - Yolanda: Gaining access to computers

by MeenaV936 Fri Jul 05, 2019 10:06 pm

I picked A because I felt that the distinction Yolanda makes is that joyriding directly causes physical harm, whereas computer hacking doesn't. Whereas Arjun doesn't maintain the distinction between directly causing harm and indirectly causing harm. He says computer hacking harms people but it is indirect harm, when Yolanda is talking about a difference in direct harm.

Can someone tell me why I'm wrong?
 
HughM388
Thanks Received: 2
Elle Woods
Elle Woods
 
Posts: 54
Joined: July 05th, 2020
 
 
 

Re: Q14 - Yolanda: Gaining access to computers

by HughM388 Tue Aug 11, 2020 6:31 pm

Aren't the pair of them hypothesizing anyway? Yolanda has already established her argument in terms of the potentiality—and not the actuality—of harm. Yolanda says, "Since joyriding COULD cause harm it IS more dangerous." Potential to actual. Arjun simply responds in kind, mirroring the structure of Yolanda's claim. "Computer hacking COULD damage data systems and therefore also IS harmful," Arjun says.

When you're conducting a conversation or debate that relies on such hypotheticals and potentialities, is it a flaw to continue that conversation or debate in that rhetorical mode?